با ادامه کار این ماراتن موافقید؟


  • مجموع رای دهندگان
    331

mojtabaaa1373

Active Member
ارسال ها
362
لایک ها
74
امتیاز
28
با استفاده از پیمانه گرفتن ومباحث مقدماتی مانند این که هر عدد مربع کامل به فرم8k+1 هست حل میشه.
 

rezashiri

Well-Known Member
ارسال ها
1,458
لایک ها
325
امتیاز
83
mojtaba-1373 گفت
وجود ندارد اگه سه رقم آخر یکی باشه باید یا همه 0 باشن یا همه 4.

اشتباه نکردید؟!؟

مربع کامل نیست ، مکعب کامله ...

مثلا یه عدد اینه : (کوچیکترینش نیستا ...) 2192
 

MR.Amin

New Member
ارسال ها
594
لایک ها
202
امتیاز
0
خب؟؟؟

ادامه اش چی؟؟
 

MR.Amin

New Member
ارسال ها
594
لایک ها
202
امتیاز
0
192 میشه جواب؟؟

فکر کنم!

درسته؟؟؟
 

Aref

New Member
ارسال ها
1,262
لایک ها
1,008
امتیاز
0
فکر کنم من این سوال رو قبلا مطرح کردم.
 

MR.Amin

New Member
ارسال ها
594
لایک ها
202
امتیاز
0
من ندیده بودم!!!

حالا جواب درسته یا نه؟؟؟
 

mojtabaaa1373

Active Member
ارسال ها
362
لایک ها
74
امتیاز
28
برای حل ابتدا به پیمانه ی 5 سپس 25 سپس 125 بگیرید و از این نکته استفاده کنید که kوk+1000 تفاوتی درسه رقم آخر ندارند.
 

AidinT

New Member
ارسال ها
431
لایک ها
22
امتیاز
0
[center:b5c8d6d9b6]38[/center:b5c8d6d9b6]معادله ی زیر را حل کنید:
[center:b5c8d6d9b6]
[/center:b5c8d6d9b6]
 

AidinT

New Member
ارسال ها
431
لایک ها
22
امتیاز
0
راه حل سوال 37 با راه حل معادله ی بالا یکیه! راه حل منم واسه سوال 37 همین معادله هست!!!!!
 

mojtabaaa1373

Active Member
ارسال ها
362
لایک ها
74
امتیاز
28
برای این سوال هم مثله حلی که برای قبلی گفتم اول به پیمانهی 5 بعد 25 بعد125 بعد250 میگیریم کلا این روش برای بدست آوردن چنین معادلاتی خیلی بدرد می خوره.
 

AidinT

New Member
ارسال ها
431
لایک ها
22
امتیاز
0
آفرین، روش کلی هم به همین شکل هست.
اول به پیمانه ی یک عدد کوچیک می گیریم، معادله رو برای اون حل می کنیم، سپس با جاگذاری جواب پیمانه رو بزرگ و بزرگ تر می کنیم.
حالا سوال بعدی

[center:57fe2c8c1a]39

[/center:57fe2c8c1a]
اگر b>1 عدد صحیح دلخواهی باشد، به طوری که
، آیا می توان نتیجه گرفت که
؟
 

fereidoon

Active Member
ارسال ها
447
لایک ها
132
امتیاز
43
بله می توان,برعکسشم میشه.
 

MR.Amin

New Member
ارسال ها
594
لایک ها
202
امتیاز
0
میتوانش رو اثبات کن!!!

ما هم میدونیم میتوان!!!
 

fereidoon

Active Member
ارسال ها
447
لایک ها
132
امتیاز
43
از اینکه 1-(a^m-1,a^n-1)=a^(m,n) استفاده کن و برای اثبات این هم یه روش نزولی بنویسی حله!


p.s:نمی دونم چرا latex قاطی کرده نمی یاد!!!!!
 

MR.Amin

New Member
ارسال ها
594
لایک ها
202
امتیاز
0
آرره!!!

[HIGHLIGHT=#7030a0] 40 [/HIGHLIGHT]
ثابت کنید:

 

fereidoon

Active Member
ارسال ها
447
لایک ها
132
امتیاز
43
من راه حل مقدماتی برای این سوال نمی دونم,اما راه حل غیر مقدماتی اون با شمردن توان p(عدد اول)از دو طرف و ثابت کردن اینکه توانp سمت چپ از سمت راست بیشتره!

p.s:چه جوری latex رو اوردی؟
 

rezashiri

Well-Known Member
ارسال ها
1,458
لایک ها
325
امتیاز
83
مربع کامل

[center:12bab67a83]
[/center:12bab67a83]
ثابت کنید اگر
آنگاه
مربع کامل است.
 

rezashiri

Well-Known Member
ارسال ها
1,458
لایک ها
325
امتیاز
83
[center:14c581c2a3]
[/center:14c581c2a3]


تمام زوج های مرتب
را از اعداد صحیح و مثبت بیابید که :

[center:14c581c2a3]
[/center:14c581c2a3]


[center:14c581c2a3]تذکر : از این به بعد هرکی یه سوال حل می کنه حتما یه سوال بذاره.[/center:14c581c2a3]
 

fereidoon

Active Member
ارسال ها
447
لایک ها
132
امتیاز
43
اینها هیچکدومشون مقدماتی نیستند!!!!
 
بالا